Last visit was: 23 Apr 2024, 12:52 It is currently 23 Apr 2024, 12:52

Close
GMAT Club Daily Prep
Thank you for using the timer - this advanced tool can estimate your performance and suggest more practice questions. We have subscribed you to Daily Prep Questions via email.

Customized
for You

we will pick new questions that match your level based on your Timer History

Track
Your Progress

every week, we’ll send you an estimated GMAT score based on your performance

Practice
Pays

we will pick new questions that match your level based on your Timer History
Not interested in getting valuable practice questions and articles delivered to your email? No problem, unsubscribe here.
Close
Request Expert Reply
Confirm Cancel
SORT BY:
Date
Tags:
Show Tags
Hide Tags
Math Expert
Joined: 02 Sep 2009
Posts: 92881
Own Kudos [?]: 618579 [53]
Given Kudos: 81562
Send PM
Most Helpful Reply
Current Student
Joined: 31 Aug 2016
Status:Valar Dohaeris
Posts: 299
Own Kudos [?]: 916 [18]
Given Kudos: 911
GMAT 1: 700 Q49 V37
Send PM
General Discussion
Manager
Manager
Joined: 20 Apr 2018
Posts: 141
Own Kudos [?]: 289 [1]
Given Kudos: 156
Concentration: Technology, Nonprofit
Schools: ISB '21 (A)
WE:Analyst (Non-Profit and Government)
Send PM
Board of Directors
Joined: 11 Jun 2011
Status:QA & VA Forum Moderator
Posts: 6072
Own Kudos [?]: 4688 [4]
Given Kudos: 463
Location: India
GPA: 3.5
WE:Business Development (Commercial Banking)
Send PM
Re: In Millington, a city of 50,000 people, Mercedes Pedrosa, a realtor [#permalink]
3
Kudos
1
Bookmarks
Bunuel wrote:
In Millington, a city of 50,000 people, Mercedes Pedrosa, a realtor, calculated that a family with Millington’s median family income, $28,000 a year, could afford to buy Millington’s median-priced $77,000 house. This calculation was based on an 11.2 percent mortgage interest rate and on the realtor’s assumption that a family could only afford to pay up to 25 percent of its income for housing.

Which of the following corrections of a figure appearing in the passage above, if it were the only correction that needed to be made, would yield a new calculation showing that even incomes below the median family income would enable families in Millington to afford Millington’s median-priced house?


(A) Millington’s total population was 45,000 people.

(B) Millington’s median annual family income was $27,000.

(C) Millington’s median-priced house cost $80,000.

(D) The rate at which people in Millington had to pay mortgage interest was only 10 percent.

(E) Families in Millington could only afford to pay up to 22 percent of their annual income for housing.


Median = Middle Value of a set....

Income : ...............................$ 28,000...............................
Price of House : ...............................$ 77,000...............................

Interest Rate : 11.2 %

The highlighted part of the question stem requires us to find the possibility for the residents to purchase House....

(A) Irrelevant.
(B) If annual Income decreases then the citizens may not be able to afford to purchase the house unless The price of the HOuse or the Interest rates decreases.
(C) If the price of the house increases then the citizens may not be able to afford to purchase the house unless the income Increases or the Interest rates decreases.
(E) Out of scope...

Hence correct Answer must be (D)
Intern
Intern
Joined: 23 Jul 2015
Posts: 8
Own Kudos [?]: 12 [2]
Given Kudos: 3
Send PM
Re: In Millington, a city of 50,000 people, Mercedes Pedrosa, a realtor [#permalink]
2
Kudos
The question asks to weaken the argument. The argument is based on:
- 11.2 percent mortgage interest rate
- family could only afford to pay up to 25 percent of its income for housing.
So any answers that talk about either lower actual mortgage interest rate or higher percent of income for housing will weaken the argument.
Only D
Verbal Forum Moderator
Joined: 08 Dec 2013
Status:Greatness begins beyond your comfort zone
Posts: 2101
Own Kudos [?]: 8805 [1]
Given Kudos: 171
Location: India
Concentration: General Management, Strategy
GPA: 3.2
WE:Information Technology (Consulting)
Send PM
Re: In Millington, a city of 50,000 people, Mercedes Pedrosa, a realtor [#permalink]
1
Kudos
In Millington, a city of 50,000 people, Mercedes Pedrosa, a realtor, calculated that a family with Millington’s median family income, $28,000 a year, could afford to buy Millington’s median-priced $77,000 house. This calculation was based on an 11.2 percent mortgage interest rate and on the realtor’s assumption that a family could only afford to pay up to 25 percent of its income for housing.

Which of the following corrections of a figure appearing in the passage above, if it were the only correction that needed to be made, would yield a new calculation showing that even incomes below the median family income would enable families in Millington to afford Millington’s median-priced house?

To buy Millington’s median-priced $77,000 house
1. Median family income of 28k $
2. 11.2 percent mortgage interest rate
3. A family could only afford to pay up to 25% of its income for housing

(A) Millington’s total population was 45,000 people. - Incorrect - we don't know how the decrease in population has affected the median values

(B) Millington’s median annual family income was $27,000. -Opposite; Fewer people will able to afford since the initial calculation is based on rule 1

(C) Millington’s median-priced house cost $80,000. - Opposite; Fewer people will be able to afford since median house price was taken as 77K $ in the calculations

(D) The rate at which people in Millington had to pay mortgage interest was only 10 percent. - Correct

(E) Families in Millington could only afford to pay up to 22 percent of their annual income for housing. - Opposite; fewer people will be able to afford because the initial calculation is based on Rule 3

Answer D
IIM School Moderator
Joined: 04 Sep 2016
Posts: 1261
Own Kudos [?]: 1238 [0]
Given Kudos: 1207
Location: India
WE:Engineering (Other)
Send PM
Re: In Millington, a city of 50,000 people, Mercedes Pedrosa, a realtor [#permalink]
DavidTutorexamPAL aragonn VeritasKarishma GMATNinja nightblade354 gmat1393

Can you help me in the understanding of argument?

Quote:
In Millington, a city of 50,000 people, Mercedes Pedrosa, a realtor, calculated that a family with Millington’s median family income, $28,000 a year, could afford to buy Millington’s median-priced $77,000 house. This calculation was based on an 11.2 percent mortgage interest rate and on the realtor’s assumption that a family could only afford to pay up to 25 percent of its income for housing.

Median family income: $28 k
Available to buy house: $7 K
Additional loan interest: 11.2%

I could not comprehend the numbers above.

Quote:
Which of the following corrections of a figure appearing in the passage above, if it were the only correction that needed to be made, would yield a new calculation showing that even incomes below the median family income would enable families in Millington to afford Millington’s median-priced house?

So, are we trying to say that above premises are incorrect? We never challenge facts in CR premises, do we?
Retired Moderator
Joined: 23 Sep 2015
Posts: 1267
Own Kudos [?]: 5650 [2]
Given Kudos: 416
Send PM
Re: In Millington, a city of 50,000 people, Mercedes Pedrosa, a realtor [#permalink]
1
Kudos
adkikani - No for sure. Given information is absolutely correct as always. For solving this question I would ask you to put urself in this situation and think if you need to purchase a house in given condition what you would do ?

Think of it Assuming a situation of when your income is $28k, you can give the above given share, 1/4 of the amount easily.
Now Let say your salary decreased to $20k. You need to purchase same house. Which means you need to make same payments. I hope you can see that now you need to pay same amount and that amount will be more than 1/4rth of the amount you have as of now.
Premise has clearly stated that ---- a family could only afford to pay up to 25 percent of its income for housing.

So now with $20k in hand how a family can buy same house.My pre-thoughts are as following.
1. Suddenly cost of house decreased and its relative to new salary. now you need to pay less amount and may be it come to your 1/4 rth limit.
2. Cause of given interest rate, One has to pay greater amount. If interest rate is somewhat less than this then this one is also possible.

I see option D is on those lines. Rest choices are not even close.
VP
VP
Joined: 09 Mar 2016
Posts: 1160
Own Kudos [?]: 1017 [3]
Given Kudos: 3851
Send PM
Re: In Millington, a city of 50,000 people, Mercedes Pedrosa, a realtor [#permalink]
3
Kudos
Bunuel wrote:
In Millington, a city of 50,000 people, Mercedes Pedrosa, a realtor, calculated that a family with Millington’s median family income, $28,000 a year, could afford to buy Millington’s median-priced $77,000 house. This calculation was based on an 11.2 percent mortgage interest rate and on the realtor’s assumption that a family could only afford to pay up to 25 percent of its income for housing.

Which of the following corrections of a figure appearing in the passage above, if it were the only correction that needed to be made, would yield a new calculation showing that even incomes below the median family income would enable families in Millington to afford Millington’s median-priced house?


(A) Millington’s total population was 45,000 people.

(B) Millington’s median annual family income was $27,000.

(C) Millington’s median-priced house cost $80,000.

(D) The rate at which people in Millington had to pay mortgage interest was only 10 percent.

(E) Families in Millington could only afford to pay up to 22 percent of their annual income for housing.



Hey adkikani, when you have a doubt which answer choice to choose, use POE i.e. PREDICT, OBSERVE, ELIMINATE - law of GMAT samurai :grin: :lol:


(A) Millington’s total population was 45,000 people. ( it is out of scope :) (talks about population)

(B) Millington’s median annual family income was $27,000. ( it is correction but for the worse :lol: )

(C) Millington’s median-priced house cost $80,000. ( we are not concerned about cost of house)

(D) The rate at which people in Millington had to pay mortgage interest was only 10 percent. :)

(E) Families in Millington could only afford to pay up to 22 percent of their annual income for housing ( this option wont work because we need to make such corrections so that family with below median salary could afford a house )


So you are left with D :grin:
Intern
Intern
Joined: 30 Aug 2018
Posts: 7
Own Kudos [?]: 6 [0]
Given Kudos: 18
GPA: 4
WE:Marketing (Pharmaceuticals and Biotech)
Send PM
Re: In Millington, a city of 50,000 people, Mercedes Pedrosa, a realtor [#permalink]
(A) Millington’s total population was 45,000 people. --> Unrelated to affordability. Incorrect.

(B) Millington’s median annual family income was $27,000. --> Same as stated. Incorrect.

(C) Millington’s median-priced house cost $80,000. --> Inverse of what is asked since Median is increased. Incorrect.

(D) The rate at which people in Millington had to pay mortgage interest was only 10 percent. --> Correct. Interest rate decreases.

(E) Families in Millington could only afford to pay up to 22 percent of their annual income for housing. --> Inverse of what is asked since affordability is decreased. Incorrect.
GMAT Club Verbal Expert
Joined: 13 Aug 2009
Status: GMAT/GRE/LSAT tutors
Posts: 6917
Own Kudos [?]: 63649 [3]
Given Kudos: 1773
Location: United States (CO)
GMAT 1: 780 Q51 V46
GMAT 2: 800 Q51 V51
GRE 1: Q170 V170

GRE 2: Q170 V170
Send PM
Re: In Millington, a city of 50,000 people, Mercedes Pedrosa, a realtor [#permalink]
1
Kudos
2
Bookmarks
Expert Reply
adkikani wrote:
DavidTutorexamPAL aragonn VeritasKarishma GMATNinja nightblade354gmat1393

Can you help me in the understanding of argument?

Quote:
In Millington, a city of 50,000 people, Mercedes Pedrosa, a realtor, calculated that a family with Millington’s median family income, $28,000 a year, could afford to buy Millington’s median-priced $77,000 house. This calculation was based on an 11.2 percent mortgage interest rate and on the realtor’s assumption that a family could only afford to pay up to 25 percent of its income for housing.

Median family income: $28 k
Available to buy house: $7 K
Additional loan interest: 11.2%

I could not comprehend the numbers above.

Quote:
Which of the following corrections of a figure appearing in the passage above, if it were the only correction that needed to be made, would yield a new calculation showing that even incomes below the median family income would enable families in Millington to afford Millington’s median-priced house?

So, are we trying to say that above premises are incorrect? We never challenge facts in CR premises, do we?

The question itself explicitly gives us permission to change a single fact in the passage. It's very similar to "if true, which of the following..." except that we're given one changed fact, as opposed to one additional fact.

As covered by PeepalTree, sandman13, cngthanh190, aragonn, and others (thanks for your explanations!), we can step back from the specific figures and ask ourselves: What kind of change would make the median-priced house affordable on a below-median income?

Quote:
(A) Millington’s total population was 45,000 people.

Quote:
(B) Millington’s median annual family income was $27,000.

Huh? (A) and (B) don't fit into this question all. Total population has nothing to do with whether a family can afford to buy a home. And changing to the median income won't help or hurt, because we're specifically concerned with what's affordable on a below-median income.

That's why we eliminate (A) and (B) immediately.

Quote:
(C) Millington’s median-priced house cost $80,000.

If we change the cost of a median-priced house, then we could definitely make it affordable to more families. But this change goes in the opposite direction of what we want! $80,000 would be less affordable than $77,000, so we'll eliminate (C).

Quote:
(D) The rate at which people in Millington had to pay mortgage interest was only 10 percent.

(D) is basically a better version of (C). This choice identifies a factor that could make houses more affordable: The amount of interest paid on the house's mortgage. And it tells us that the amount of interest is going down. Since nothing else in the passage changes, this would result in more families being able to afford the median priced house -- even if they make less than the median income of $28,000 per year.

Quote:
(E) Families in Millington could only afford to pay up to 22 percent of their annual income for housing.

Like (C), this choice identifies a variable that determines what families can afford to pay, but it goes in the wrong direction. Since 22% is a lower ceiling than 25%, making this one change would mean poorer families have a harder time buying median-priced house.

I hope this shows how to tackle the question and eliminate answer choices without doing any arithmetic. :)
Manager
Manager
Joined: 16 May 2016
Posts: 119
Own Kudos [?]: 369 [0]
Given Kudos: 124
Location: India
Concentration: Marketing, International Business
GMAT 1: 720 Q50 V38
GPA: 3.5
WE:Analyst (Consulting)
Send PM
In Millington, a city of 50,000 people, Mercedes Pedrosa, a realtor [#permalink]
Bunuel wrote:
In Millington, a city of 50,000 people, Mercedes Pedrosa, a realtor, calculated that a family with Millington’s median family income, $28,000 a year, could afford to buy Millington’s median-priced $77,000 house. This calculation was based on an 11.2 percent mortgage interest rate and on the realtor’s assumption that a family could only afford to pay up to 25 percent of its income for housing.

Which of the following corrections of a figure appearing in the passage above, if it were the only correction that needed to be made, would yield a new calculation showing that even incomes below the median family income would enable families in Millington to afford Millington’s median-priced house?


(A) Millington’s total population was 45,000 people.

(B) Millington’s median annual family income was $27,000.

(C) Millington’s median-priced house cost $80,000.

(D) The rate at which people in Millington had to pay mortgage interest was only 10 percent.

(E) Families in Millington could only afford to pay up to 22 percent of their annual income for housing.

Easy Question.
Here is my analysis:
If we want to prove that families can afford a house of $77000 even if the median income is $28000, we need to adjust the rate of interest placed on the mortgage. And for lower income rate should be lower. In that sense, D is the only answer.­
Manager
Manager
Joined: 22 Sep 2018
Posts: 73
Own Kudos [?]: 75 [0]
Given Kudos: 95
Send PM
Re: In Millington, a city of 50,000 people, Mercedes Pedrosa, a realtor [#permalink]
As the question asks for a correction , that need to be made so that, people below the median level can purchase the house,
the option which says that the less than 28,000 is also fine is the answer,

Among all the options , options D says that the interstate's is 10% In actually , keeping all same then reducing the interest mean reducing the money that need to be paid, so few more who are less than 28,000 will also come into the affordable category.
Intern
Intern
Joined: 02 May 2020
Posts: 39
Own Kudos [?]: 20 [0]
Given Kudos: 18
Send PM
In Millington, a city of 50,000 people, Mercedes Pedrosa, a realtor [#permalink]
Bunuel wrote:
In Millington, a city of 50,000 people, Mercedes Pedrosa, a realtor, calculated that a family with Millington’s median family income, $28,000 a year, could afford to buy Millington’s median-priced $77,000 house. This calculation was based on an 11.2 percent mortgage interest rate and on the realtor’s assumption that a family could only afford to pay up to 25 percent of its income for housing.

Which of the following corrections of a figure appearing in the passage above, if it were the only correction that needed to be made, would yield a new calculation showing that even incomes below the median family income would enable families in Millington to afford Millington’s median-priced house?


(A) Millington’s total population was 45,000 people.

(B) Millington’s median annual family income was $27,000.

(C) Millington’s median-priced house cost $80,000.

(D) The rate at which people in Millington had to pay mortgage interest was only 10 percent.

(E) Families in Millington could only afford to pay up to 22 percent of their annual income for housing.


Reading critically,
The question stem states, "Which of the following corrections of a figure appearing in the passage above, if it were the only correction that needed to be made, would yield a new calculation showing that even incomes below the median family income would enable families in Millington to afford Millington’s median-priced house?

The focus here is on changing the calculation parameters. Now we look at a specific portion of the stimuli, " This calculation was based on an 11.2 percent mortgage interest rate and on the realtor’s assumption that a family could only afford to pay up to 25 percent of its income for housing."

The only two parameters talking about calculation are Mortgage interest rates and Percentage of income that the families can afford to pay for housing.

For the answer, we basically need to change one of the parameters to make the housing more affordable.

Choices 'D' and 'E' refer to the variables of the calculation and hence are are only two competitors.

e - Lesser percentage of income to be contributed for housing - Would mean lesser buying power and hence this would have an inverse effect.
d - Mortgage rates reduced from 11.2 to 10. This would definitely make housing more affordable and hence is the correct answer.­
User avatar
Non-Human User
Joined: 01 Oct 2013
Posts: 17206
Own Kudos [?]: 848 [0]
Given Kudos: 0
Send PM
Re: In Millington, a city of 50,000 people, Mercedes Pedrosa, a realtor [#permalink]
Hello from the GMAT Club VerbalBot!

Thanks to another GMAT Club member, I have just discovered this valuable topic, yet it had no discussion for over a year. I am now bumping it up - doing my job. I think you may find it valuable (esp those replies with Kudos).

Want to see all other topics I dig out? Follow me (click follow button on profile). You will receive a summary of all topics I bump in your profile area as well as via email.
GMAT Club Bot
Re: In Millington, a city of 50,000 people, Mercedes Pedrosa, a realtor [#permalink]
Moderators:
GMAT Club Verbal Expert
6917 posts
GMAT Club Verbal Expert
238 posts
CR Forum Moderator
832 posts

Powered by phpBB © phpBB Group | Emoji artwork provided by EmojiOne